LSAT and Law School Admissions Forum

Get expert LSAT preparation and law school admissions advice from PowerScore Test Preparation.

 Administrator
PowerScore Staff
  • PowerScore Staff
  • Posts: 8917
  • Joined: Feb 02, 2011
|
#41417
Please post your questions below!
 lathlee
  • Posts: 652
  • Joined: Apr 01, 2016
|
#46175
Hi. I thought E) was the better answer choice than b) considering that assumption questions, usually, avoid the strongly worded answer choices are better off. I don't understand why E) is incorrect over b)
 BostonLawGuy
  • Posts: 52
  • Joined: Jul 13, 2018
|
#57762
Hi Lathlee,

I note that you have not received a response, so I thought I would offer my take on this.

Conclusion: Doctors should be prepared to discuss yoga with patients.
Premises: If an activity significantly reduces lower back pain, doctors should discuss with patients
Premies: Yoga decreases lower back pain just as much as stretching with a PT

It must be true, if the argument is valid that stretching with a PT significantly reduces lower back pain.

Try the negation test: Stretching with a PT does not significantly lower back pain. (Exaggerating it makes it more obvious: stretching with a PT doesn't help at all, sometimes it actually makes it worse!)

If the author is equating yoga with stretching and implying that yoga significantly reduces lower back pain, then stretching has to significantly reduce lower back pain. Otherwise the author could be comparing yoga to something that is not effective at all. Kinda like me saying that I'm just as smart as that nit-wit. LOL. Can't really say I am "significantly smart" if I'm comparing myself to a dumb-ass!

Which of the two statements demonstrates that I am "strong?" I can lift just as much weight as that superhero! OR: I can lift just as much weight as that 10 year-old child.

Interestingly, this same reasoning was used in test 37 Sec. 2 Q 19. Take a look at that one.
 Adam Tyson
PowerScore Staff
  • PowerScore Staff
  • Posts: 5153
  • Joined: Apr 14, 2011
|
#60851
Good analysis, Boston, and thanks for the hand!

The problem with E is based on a Mistaken Reversal. To paraphrase a bit, if a treatment is effective, doctors should talk about it. Answer E relies on a reversal of that claim - if doctors talk about it, it is effective. Doctors talking about stretching doesn't mean stretching is effective, and yoga being just as good as something that may not be effective doesn't mean that doctors should talk about yoga. We need to know that yoga is effective, and we get there by proving that stretching is effective (since yoga is just as effective).

Get the most out of your LSAT Prep Plus subscription.

Analyze and track your performance with our Testing and Analytics Package.